How do I solve an indefinite integral with a trigonometric substitution?

  • Thread starter Thread starter mat331760298
  • Start date Start date
  • Tags Tags
    Integral
Click For Summary
To solve the indefinite integral of 1/[(x^4)(sqrt(9x^2 -1))], a trigonometric substitution is applied using x = 3sec(y), leading to dx = 3sec(y)tan(y)dy. This transforms the integral into a more manageable form, involving secant and tangent functions. The discussion confirms that after substitution, the integral simplifies to 27*cos^3(t) dt, which can be integrated to yield 27(sin(t) - (1/3)(sin(t))^3). Finally, the antiderivative is obtained, and the original substitution is reversed to complete the solution. This method effectively demonstrates the use of trigonometric identities in solving complex integrals.
mat331760298
Messages
13
Reaction score
0
1. indefinite integral of 1/[(x^4)(sqrt(9x^2 -1))]

I'm looking for step by step help here. I use x=3secy and dx=3secytanydy to convert the top function to integral of: [3secytanydy]/[(tany)(3secy)^4]

I am not sure if this is right/how to get further
 
Physics news on Phys.org


mat331760298 said:
1. indefinite integral of 1/[(x^4)(sqrt(9x^2 -1))]

I'm looking for step by step help here. I use x=3secy and dx=3secytanydy to convert the top function to integral of: [3secytanydy]/[(tany)(3secy)^4]

I am not sure if this is right/how to get further

I labelled by triangle with hypotenuse = 3x, adjacent side = 1, opp. side = sqrt(9x^2 - 1), with angle t.

So sec t = 3x ==> x = 1/3 sec t
dx = 1/3 sec t * tan t * dt
sqrt(9x^2 -1) = tan t

Then,
\int \frac{dx}{x^4 \sqrt{9x^2 - 1}} = \int \frac{1/3 sec(t)tan(t)dt}{(1/3)^4 sec^4(t)~tan(t)}
Can you continue from there?
 


do you get integral of:27*(cost)^3 dt ?
 


then you integrate to get 27(sint-(1/3)(sint)^3) and use opp side over hypotenuse to replace with sint?
 


Yes. After you get the antiderivative, undo your substitution, and you'll be done.
 
Question: A clock's minute hand has length 4 and its hour hand has length 3. What is the distance between the tips at the moment when it is increasing most rapidly?(Putnam Exam Question) Answer: Making assumption that both the hands moves at constant angular velocities, the answer is ## \sqrt{7} .## But don't you think this assumption is somewhat doubtful and wrong?

Similar threads

Replies
3
Views
2K
  • · Replies 105 ·
4
Replies
105
Views
6K
  • · Replies 6 ·
Replies
6
Views
2K
Replies
10
Views
2K
  • · Replies 12 ·
Replies
12
Views
2K
Replies
7
Views
2K
  • · Replies 2 ·
Replies
2
Views
1K
  • · Replies 4 ·
Replies
4
Views
2K
  • · Replies 22 ·
Replies
22
Views
2K
  • · Replies 22 ·
Replies
22
Views
3K